Site icon First Aid Team

USMLE-Rx Step 1 Qmax Challenge #4091

Check out today’s Step 1 Qmax Question Challenge.

Know the answer? Post it below! Don’t forget to check back for an update with the correct answer and explanation (we’ll post it in the comments section below).

A 70-year-old man complains of a long history of pain in his ankles, toes, and fingers. He has experienced intermittent acute attacks of exquisite pain every few months, followed by completely asymptomatic periods. In recent years, he has had near-constant discomfort at baseline, and now has permanent “swelling” in many of the joints of his fingers and toes. Joint fluid is aspirated, and the fluid is examined under polarized light (shown in the image). Treatment is initiated, and the patient’s uric acid levels gradually fall.

What is the mechanism of action of the best treatment?

A. Binds tubulin
B. Blocks formation of prostaglandins and thromboxane from arachidonic acid
C. Inhibits release of phospholipase A2
D. Inhibits xanthine oxidase
E. Selective, competitive angiotensin II receptor inhibition

———————–

Want to know the ‘bottom line?’ Purchase a USMLE-Rx Subscription and get many more features, more questions, and passages from First Aid, including images, references, and other facts relevant to this question.

This practice question is an actual question from the USMLE-Rx Step 1 test bank. For more USMLE Step 1 prep, subscribe to our Flash Facts and Step 1 Express video series. Score the best deal on all three products with a Step 1 Triple Play Bundle.

Exit mobile version